Administración     

Olimpiadas de Matemáticas
Página de preparación y problemas

Selector
La base de datos contiene 1154 problemas y 775 soluciones.
OME Local
OME Nacional
OIM
OME Andalucía
Retos UJA
Inicio
—20
—5
Problema 1152
Se considera $f(x)=x^{1997}-x+1$. Sea $n\gt 1$ un número entero. Demostrar que, para todo número entero $x$, los números $f(x)$ y $f^n(x)$ son primos entre sí.

Nota: $f^2(x)=f(f(x))$, $f^3(x)=f(f^2(x))=f(f(f(x)))$ y, en general, \[f^n(x)=f(f^{n-1}(x))=f(f(\ldots f(x))\ldots))\quad (n\text{ veces}).\]

pistasolución 1info
Pista. Observa que $f^n(x)$ es un polinomio con coeficientes enteros y término independiente igual a $1$.
Solución. Observemos que $f^n(x)$ es un polinomio con coeficientes enteros y vamos a probar que su término independiente es $1$. Esto se prueba fácilmente por inducción sobre $n$, teniendo en cuenta que el término independiente se obtiene evaluando en $x=0$. Para $n=1$, tenemos que $f^1(0)=f(0)=1$; supuesto cierto que $f^n(0)=1$ para cierto $n\geq1$, podemos calcular $f^{n+1}(0)=f(f^n(0))=f(1)=1$.

Esto nos dice que, para cada $n\in\mathbb{N}$ podemos expresar $f^{n-1}(x)=x p_{n-1}(x)+1$ para cierto polinomio $g_{n-1}(x)$. Tenemos así que \begin{align*} \mathrm{mcd}(f(x),f^n(x))&=\mathrm{mcd}(f(x),f^{n-1}(f(x)))\\ &=\mathrm{mcd}(f(x),f(x) p_{n-1}(f(x))+1)=1. \end{align*}

Nota. El resultado es también cierto cambiando $f(x)=x^{1997}-x+1$ por cualquier polinomio $f(x)$ con coeficientes enteros y $f(0)=f(1)=1$. También es cierto que $f^n(y)$ y $f^m(y)$ son primos relativos para cualesquier $m,n\in\mathbb{N}$ (basta aplicar el enunciado a $x=f^{m-1}(y)$ con $n+1$ en lugar de $n$).

Si crees que el enunciado contiene un error o imprecisión o bien crees que la información sobre la procedencia del problema es incorrecta, puedes notificarlo usando los siguientes botones:
Informar de error en enunciado Informar de procedencia del problema
Problema 1147
Se consideran los puntos del plano $P_1=(1,1000)$ , $P_2=(2,1000)$,... $P_{1998} = (1998,1000)$ y el origen de coordenadas $O = (0,0)$. Para cada uno de los puntos $P_k$ se traza el segmento $OP_k$ únicamente si no contiene más puntos con ambas coordenadas enteras que $O$ y $P_k$. ¿Cuántos segmentos se dibujan?
pistasolución 1info
Pista. Demuestra que el segmento $OP_k$ contiene un punto de coordenadas enteras si, y solamente si, $k$ y $1000$ tienen un factor común mayor que $1$.
Solución. Consideremos un punto $P=(a,b)$ en general con sus dos coordenadas enteras no negativas, es decir, $a,b\in\mathbb{N}$. Vamos a demostrar previamente que el segmento $OP$ contiene otro punto de coordenadas enteras si, y sólo si, $a$ y $b$ no son primos relativos.

Por un lado, si existe tal punto en $OP$, esto quiere decir que existen $c,d\in\mathbb{N}$ y $0\lt\lambda\lt 1$ tales que $(c,d)=\lambda(a,b)$. En particular, $\lambda=\frac{c}{a}=\frac{d}{b}$ es un número racional. Supongamos que $\lambda=\frac{m}{n}$ como fracción irreducible, luego \[c=\frac{m}{n}a,\qquad d=\frac{m}{n}b,\] de forma que $n$ debe ser un divisor común a $a$ y $b$. Observemos que $n\gt 1$ ya que $0\lt \lambda\lt 1$, luego hemos probado que $a$ y $b$ no son primos entre sí. Recíprocamente, si $n\gt 1$ es un divisor común a $a$ y $b$, entonces el punto $(c,d)=(\frac{a}{n},\frac{b}{n})$ es un punto de coordenadas enteras en el segmento $OP$, lo que concluye la demostración.

Con esta información, el problema se traduce en ver cuántos números enteros $k$ entre $1$ y $1998$ son primos relativos con $1000$. Como $1000=2^3\cdot 5^3$, estamos buscando los valores de $k$ que no tienen factores primos $2$ ni $5$. De los $1998$ números considerados, hay $999$ múltiplos de $2$, $399$ múltiplos de $5$ y $199$ múltiplos de $10$ (¿sabrías contarlos rápidamente?). Por tanto, la cantidad de primos relativos con $1000$ (la solución al problema) es: \[1998-999-399+199=799\] (hay que añadir $199$ ya que estamos quitando los múltiplos de $10$ dos veces).

Si crees que el enunciado contiene un error o imprecisión o bien crees que la información sobre la procedencia del problema es incorrecta, puedes notificarlo usando los siguientes botones:
Informar de error en enunciado Informar de procedencia del problema
Problema 1142
¿Existe algún número entero mayor que $10$ que sea un cuadrado perfecto y además tenga todas sus cifras iguales?
pistasolución 1solución 2info
Pista. Discutir por separado si dicho cuadrado puede estar formado por unos, doses, treses,... y nueves. Fijarse en las dos últimas cifras que puede tener un cuadrado también puede ser bastante útil.
Solución. Supongamos que $n$ es un cuadrado perfecto con todas sus cifras iguales y llamemos $a$ a la cifra de las unidades de $n$. Por un lado, si $n$ tiene todas sus cifras iguales, entonces $n=ab$ siendo $b$ un número formado sólo por unos. Por otro lado, los únicos valores posibles de $a$ son $0$, $1$, $4$, $5$, $6$ y $9$ (ya que depende sólo de la cifra de las unidades del número del que $n$ es cuadrado).
  • No puede ser $a=0$ porque entonces sería $n=0\cdot b=0<10$.
  • No puede ser $a=5$ porque entonces $b$ tendría otro factor $5$, pero claramente $b$ no es múltiplo de $5$.
  • No puede ser $a=6$ por la misma razón ($b$ tendría otro factor $2$ pero no es un número par).

En el resto de casos $a=1$, $a=4$ y $a=9$, el propio $a$ es un cuadrado perfecto, luego tendremos que ver que el número $b$ formado sólo por unos no lo es. Por reducción al absurdo, si $b=m^2$ fuera un cuadrado perfecto, entonces la cifra de las unidades de $m$ será $1$ o $9$, luego $m=10k+1$ o bien $m=10k+9$ para cierto $k\geq 1$. Elevando al cuadrado tenemos que \[b=(10k+1)^2=100k^2+20k+1=20(5k^2+k)+1,\] luego $b$ es un múltiplo de $20$ más $1$, es decir, la cifra de las decenas de $b$ es par, lo que contradice que $b$ está formado sólo por unos. De la misma forma, \[(10k+9)^2=100k^2+180k+81=20(5k^2+9k+4)+1\] no puede estar formado sólo por unos.

Solución. Las dos últimas cifras de $n^2$ dependen exclusivamente de las dos últimas cifras de $n$. Esto se debe a que, si $n=100k+p$, donde $0\leq p\leq 99$ representa a las dos últimas cifras de $n$, entonces $n^2=100(100k^2+2kp)+p^2$.

Los únicos números naturales menores que $100$ cuyos cuadrados tienen repetida las cifras de las unidades y las decenas (y son no nulas) son $12$, $38$, $62$ y $88$, que cumplen que $12^2=144$, $38^2=1444$, $62^2=3844$ y $88^2=7744$. Hemos reducido el problema a buscar los números naturales $m$ tales que $m^2=44\ldots4=4\cdots 11\ldots1$. Esto exige que $\frac{m}{2}$ sea impar (ya que el cuadrado de un número par es par). Podemos escribir $\frac{m}{2}=2l+1$ para cierto número $l$, de donde $(2l+1)^2=11\ldots1$ o bien $4l(l+1)=11\ldots10$. Esto no es posible porque los múltiplos de $4$ tienen sus dos últimos dígitos $00$ o múltiplo de $4$, pero $10$ no es múltiplo de $4$.

Nota. Esta es una solución aportada por Samuel Gómez Moreno.

Si crees que el enunciado contiene un error o imprecisión o bien crees que la información sobre la procedencia del problema es incorrecta, puedes notificarlo usando los siguientes botones:
Informar de error en enunciado Informar de procedencia del problema
Problema 1138
Sophie está apoyada sobre una mesa circular y recibe un WhatsApp en el que se indica un número positivo $\ell_1$ junto con el mensaje ``Desplázate alrededor de la mesa, a izquierda o derecha y tantas veces como quieras, una distancia $\ell_1$ y verás cómo aparece un reloj''. Muerta de curiosidad, decide desplazarse a lo largo del borde de la mesa la distancia $\ell_1$ (que supone más de la mitad del perímetro de la mesa), y después la misma distancia $\ell_1$, y así sucesivamente, hasta darse cuenta de que siempre llega a los mismos $12$ puntos del borde de la mesa. A continuación, Sophie recibe otro WhatsApp con otro número $\ell_2$, mayor que el anterior y menor que el perímetro de la mesa, al que sigue un mensaje similar al primero. Vuelve a probar y se desplaza esta vez una distancia $\ell_2$ a lo largo del borde de la mesa y procede como antes hasta comprobar que también esta vez el mensaje es cierto y que siempre llega a los mismos 12 puntos del borde de la mesa.

A partir de los valores $\ell_1$ y $\ell_2$, ¿puede calcular Sophie el área de la superficie de la mesa? En caso afirmativo, indica cómo hacerlo.

pistasolución 1info
Pista. Demuestra en primer lugar que los 12 puntos están equiespaciados a lo largo del borde de la mesa. Usa después aritmética modular para modelar el problema.
Solución. Sabemos que sólo se pueden alcanzar $12$ puntos, pongamos que se llaman $p_1,\ldots,p_{12}$ y que están ordenados en sentido de las agujas del reloj. En primer lugar, hay que observar que, a partir de cualquier punto $p_i$, avanzando un cierto número de veces una distancia $\ell_1$ se llega a cualquier otro $p_j$. Esto se deduce de que repetir dos veces la elección que ha generado los 12 puntos tiene que pasar otra vez por todos ellos. En segundo lugar, vamos a probar que los puntos están equiespaciados por reducción al absurdo, tomando dos puntos consecutivos $p_k$ y $p_{k+1}$ que definen el menor de los $12$ arcos en que $p_1,\ldots,p_{12}$ dividen a la circunferencia. Si $p_j$ y $p_{j+1}$ definieran un arco mayor, lo único que hay que hacer es, una vez estemos en $p_j$, repetir el mismo número de avances de longitud $\ell_1$ que llevan de $p_k$ a $p_{k+1}$: esto nos llevará de $p_j$ a un punto $p'_j$ que está estrictamente entre $p_j$ y $p_{j+1}$, lo que nos da la contradicción buscada.

Podemos entonces identificar el vértice $p_k$ con el número $k$ y $\ell_1$ y $\ell_2$ con enteros $6\lt\ell_1\lt \ell_2\lt 12$ tales que avanzar $\ell_i$ desde $p_k$ se corresponde con sumar $k+\ell_i$ módulo $12$. Los únicos números $\ell_1$ y $\ell_2$ que permiten pasar por los $12$ puntos son los primos relativos con $12$, lo que nos dice necesariamente que $\ell_1=7$ y $\ell_2=11$. Tenemos así que el radio de la mesa $r$ verifica $\ell_1=\frac{7}{12}\cdot 2\pi r$, lo que nos da $r=\frac{6\ell_1}{7\pi}$ y nos permite calcular su área a partir del dato $\ell_1$ que conoce Sophie: \[A=\pi r^2=\frac{36\pi\,\ell_1^2}{49}.\]

Nota. En realidad, no es necesario que se envíe el segundo Whatsapp puesto que, una vez se dibujan los 12 puntos, Sophie puede demostrar que son equidistantes con el argumento dado, y después sabe que avanzar la distancia $\ell_1$ supone 7 posiciones (porque ella puede contarlas, aunque nosotros no tengamos ese dato, es decir, ella sabe distinguir si avanza 7 u 11 posiciones).

Si crees que el enunciado contiene un error o imprecisión o bien crees que la información sobre la procedencia del problema es incorrecta, puedes notificarlo usando los siguientes botones:
Informar de error en enunciado Informar de procedencia del problema
Problema 1134
¿Es posible encontrar $2024$ números enteros positivos distintos tales que ninguno es un cuadrado perfecto y al sumar dos o más de ellos tampoco se obtiene un cuadrado perfecto?
pistasolución 1solución 2info
Pista. Piensa en qué ocurre si vamos eligiendo uno a uno los enteros de forma que el $n$-ésimo es un cuadrado muchísimo más grande que los $n-1$ anteriores.
Solución. Vamos a probar que se puede encontrar cualquier cantidad de enteros positivos cumpliendo esta propiedad, lo que nos da una respuesta afirmativa a la pregunta. Concretamente, vamos a demostrar que si tenemos $x_1,x_2,\ldots,x_n$ enteros positivos distintos tales que al sumar dos o más de ellos nunca se obtiene un cuadrado perfecto, entonces podemos añadir $x_{n+1}$ mayor que todos ellos y se sigue cumpliendo la propiedad (esto puede escribirse también como una demostración por inducción). Si observamos que entre dos cuadrados consecutivos $k^2$ y $(k+1)^2$ hay una diferencia de $2k+1$, bastará tomar $x_{n+1}=k^2+1$ mayor que $x_1,x_2,\ldots,x_n$ y tal que $2k\gt x_1+x_2+\ldots+x_n$.

Para terminar, justificaremos que esta elección cumple lo que queremos. Al sumar dos o más de los números $x_1,x_2,\ldots,x_{n+1}$, si ninguno de ellos es $x_{n+1}$, ya tenemos que su suma no es un cuadrado perfecto (habíamos supuesto que $x_1,\ldots,x_n$ cumplen la propiedad). Por el contrario, si $x_{n+1}$ es uno de los números elegidos, entonces la suma será $k^2+1$ más otro número que es como mucho $x_1+x_2+\ldots+x_n\lt 2k$, es decir, la suma estará entre $k^2$ y $(k+1)^2$, luego no puede ser un cuadrado.

Solución. Tomamos $2024$ potencias impares distintas de un primo $p$ (ninguna de ellas es, por tanto, un cuadrado perfecto). Si sumamos cualquier número de ellas, podemos sacar factor común la más pequeña, que multiplica a $1$ más una serie de potencias de $p$. Por tanto, dicha suma será una potencia impar de $p$ que multiplica a un número que es de la forma $kp+1$. Deducimos así que no se trata de un cuadrado (el exponente de $p$ debería ser par para que fuera un cuadrado perfecto).
Si crees que el enunciado contiene un error o imprecisión o bien crees que la información sobre la procedencia del problema es incorrecta, puedes notificarlo usando los siguientes botones:
Informar de error en enunciado Informar de procedencia del problema
José Miguel Manzano © 2010-2024. Esta página ha sido creada mediante software libre